Regional 2015 N1 P2

Problemas que aparecen en el Archivo de Enunciados.
Avatar de Usuario
Matías V5

Colaborador-Varias OFO - Jurado-OFO 2015 OFO - Jurado-OFO 2016 FOFO 6 años - Jurado-FOFO 6 años OFO - Jurado-OFO 2017
OFO - Jurado-OFO 2018 OFO - Jurado-OFO 2020 OFO - Jurado-OFO 2021
Mensajes: 1114
Registrado: Dom 17 Oct, 2010 4:44 pm
Medallas: 8
Nivel: Exolímpico

Regional 2015 N1 P2

Mensaje sin leer por Matías V5 »

Inicialmente, en el pizarrón, está escrito el número $1$. Hay dos operaciones permitidas que pueden elegirse a voluntad.
  1. Escribir debajo del último número escrito, ese número multiplicado por $2$.
  2. Escribir debajo del último número escrito, ese número cambiándole el orden a sus dígitos.
    No está permitido que el nuevo número comience con el dígito $0$.
Decidir si es posible, después de aplicar varias veces operaciones permitidas, obtener:
  1. el número $10^9=1000000000$;
  2. el número $9876543210$.
En caso afirmativo, dar la sucesión de operaciones y en caso negativo, explicar por qué es imposible.
We gave you a start so you'd know what to do
You've seen how it works, now it's over to you (...)
For there's so much more to explore!

Numberblocks - https://www.youtube.com/watch?v=KzTR72_srTU
Anna
Mensajes: 8
Registrado: Lun 28 Sep, 2015 5:43 pm
Nivel: 2
Ubicación: Azul, Buenos Aires
Contactar:

Re: Regional 2015 N1 P2

Mensaje sin leer por Anna »

Tenes las soluciones??????? Gracias!!
jujumas

OFO - Mención-OFO 2015 OFO - Medalla de Plata-OFO 2016 FOFO 6 años - Medalla Especial-FOFO 6 años OFO - Oro perfecto-OFO 2017 FOFO Pascua 2017 - Medalla-FOFO Pascua 2017
FOFO 7 años - Medalla Especial-FOFO 7 años OFO - Medalla de Oro-OFO 2018 FOFO 8 años - Jurado-FOFO 8 años OFO - Jurado-OFO 2019 FOFO Pascua 2019 - Jurado-FOFO Pascua 2019
FOFO 9 años - Jurado-FOFO 9 años OFO - Jurado-OFO 2020 COFFEE - Jurado-COFFEE Ariel Zylber
Mensajes: 402
Registrado: Dom 26 Oct, 2014 8:30 pm
Medallas: 13
Nivel: Exolímpico

Re: Regional 2015 N1 P2

Mensaje sin leer por jujumas »

Comentario sobre el problema:
Spoiler: mostrar
Para mí este fue el problema más jodido de la prueba, ya que necesitabas bastante suerte para encontrar el ejemplo para i) y la idea que se usaba para demostrar ii) era bastante rebuscada y era muy probable que no se te llegue a ocurrir (se me ocurrió la solución literalmente a 40 minutos de terminar la prueba).
Mi solución:
Spoiler: mostrar
i) Observemos primero que [math] y que con estos dígitos podemos armar [math], que es la octava parte de mil. Luego, acabamos de ver que es posible llegar de [math] a [math]. Para llegar a [math] entonces, podemos volver a hacer este proceso dos veces más. Multiplicamos [math] por [math] nueve veces hasta que nos quede [math], invertimos los dígitos a [math], multiplicamos por [math] tres veces hasta llegar a [math], multiplicamos [math] por [math] nueve veces, llegando a [math], invertimos de nuevo, y multiplicando por [math] tres veces llegamos a [math], que es lo que esta parte pedía.

ii) Vamos a demostrar que no es posible lograr lo pedido. Llegar desde [math] hasta [math], equivale a llegar desde [math] a [math], solo que invirtiendo la operación a) (dividir por 2 en vez de multiplicar por 2). Para demostrar que es imposible llegar del número pedido a [math] voy a demostrar que todos los números por los que se pase en el procedimiento inverso son necesariamente múltiplos de [math], y que como [math] no lo es, es imposible lograr lo que el problema pide.
Notemos primero que [math] es múltiplo de [math], ya que la suma de sus dígitos también lo es. Esto quiere decir que su descomposición en primos incluye a [math]. Veamos que al aplicar la operación a), estamos sacando un [math] de la descomposición en primos de un número, por lo que si el número antes de aplicar la operación a) es múltiplo de [math], el número que quede va a seguir siéndolo necesariamente. Veamos ahora que al aplicar la operación b), la suma de dígitos del número va a seguir siendo la misma por más que el mismo cambie (ya que los dígitos son los mismos solo que en otro orden). Esto quiere decir, que si la suma de los dígitos de un número antes de que se le aplique la operación b) es múltiplo de [math] (lo que implicaría que el número fuese múltiplo de [math]), el número que salga tras aplicar la operación va a seguir teniendo suma de dígitos divisible por [math], y en consecuencia va a seguir siendo divisible por [math].
Con esto notamos que aplicándole operaciones permitidas a cualquier múltiplo de [math] nunca vamos a llegar a un número que no sea múltiplo [math], por lo que no es posible llegar desde [math] a ningún múltiplo de [math], y en particular, es imposible llegar a [math].
Comentario sobre mi solución:
Spoiler: mostrar
La idea que usé en ii) para demostrar que ningún número por los que el procedimiento pueda pasar va a ser múltiplo de [math] también se puede aplicar con múltiplos de [math], por lo que el problema te podía pedir tranquilamente que demuestres que era imposible para cualquier otro múltiplo de [math]. Que hayan usado particularmente [math] fue algo que me distrajo mucho, ya que encaré el problema viendo que no había dígitos repetidos en ese número, y no que era múltiplo de [math].
Última edición por jujumas el Jue 01 Oct, 2015 11:22 pm, editado 2 veces en total.
1  
Avatar de Usuario
Emerson Soriano

OFO - Mención-OFO 2015 OFO - Medalla de Oro-OFO 2016 OFO - Medalla de Plata-OFO 2017 OFO - Medalla de Bronce-OFO 2018 OFO - Mención-OFO 2020
OFO - Medalla de Plata-OFO 2022
Mensajes: 826
Registrado: Mié 23 Jul, 2014 10:39 am
Medallas: 6

Re: Regional 2015 N1 P2

Mensaje sin leer por Emerson Soriano »

Para la parte [math]:
Spoiler: mostrar
El paso [math] será multiplicar por [math], y el paso [math] será ordenar los dígitos. Note que si realizamos al número [math] nueve veces el paso [math], entonces obtenemos [math]. Luego, viene: [math]. Si obviamos estos tres últimos ceros para poder aplicarle a ese [math] lo mismo del anterior, entonces obtendremos [math]. Luego, obviamos los seis últimos ceros y hacemos lo mismo con el [math], para así obtener [math].
Para la parte [math]:
Spoiler: mostrar
Note si un número [math] no es múltiplo de [math], entonces al multiplicarlo por [math] o al reordenar sus dígitos, jamás se obtendrá un múltiplo de [math]. Por lo tanto, como [math] no es múltiplo de [math], entonces nunca podremos obtener el número [math], ya que éste último es múltiplo de [math].
Avatar de Usuario
julianferres_

OFO - Medalla de Bronce-OFO 2015 OFO - Medalla de Plata-OFO 2016 OFO - Medalla de Plata-OFO 2017 OFO - Mención-OFO 2021
Mensajes: 388
Registrado: Sab 17 Sep, 2011 8:01 pm
Medallas: 4
Nivel: Exolímpico
Ubicación: Villa Ramallo, Buenos Aires

Re: Regional 2015 N1 P2

Mensaje sin leer por julianferres_ »

Para que sea más fácil el criterio se cumple con 3 además de con 9
1  
carlosgastont
Mensajes: 5
Registrado: Mar 22 Sep, 2015 1:15 am
Nivel: Otro

Re: Regional 2015 N1 P2

Mensaje sin leer por carlosgastont »

Para mi se podría justificar así:
987654210 es divisible por 2 y por 3 cualquier intercambio de díg que hagamos si es div por 2 lo seguirá siendo por 3. Por lo tanto es divisible por 2x3 =6.
Si llamamos X al número inicial tenemos X:(2x3) mientras dividimos solo por dos el número que quede siempre será divisible por 3 por lo tanto en algún momento ya no tendremos ningún díg como el 2, 0,6,8, 4 para poner al final de modo que sea div por 2. Por lo tanto no podríamos obtener del X inicial nunca un 1 solo div por.dos y reordenando dígitos.
1  
Avatar de Usuario
FelipeGigena

OFO - Mención-OFO 2023 OFO - Mención-OFO 2024
Mensajes: 27
Registrado: Vie 13 May, 2022 8:29 pm
Medallas: 2
Nivel: 2

Re: Regional 2015 N1 P2

Mensaje sin leer por FelipeGigena »

Spoiler: mostrar
i)

Comenzando en $1 (2^0)$, podemos ir multiplicando por 2 hasta $2^9$ que es $512$. Luego, podemos reordenar los dígitos de $512$ para obtener $125$.
Dado el $125$, lo vamos a seguir multiplicando por $2$:

$125 * 2 = 250$
$250 * 2 = 500$
$500 * 2 = 1000$

Al llegar a 1000 podemos ver que los resultados serán similares a los que nos dio el 1, salvo que vamos a tener más ceros:

$1000 * 2^9 = 512000$

De nuevo, podemos girar los dígitos y nos queda $125000$.
Ya sólo nos queda repetir el proceso:

$125000 * 2^3 = 1000000$
$1000000 * 2^9 = 512000000$
$512000000$ $\to$ $125000000$

$125000000 * 2^3 = 1000000000 = 10^9$
Spoiler: mostrar
ii)
Notemos que cada vez que reordenamos los dígitos de un número, este siempre mantiene su resto en módulo $3$. Esto pasa porque el resto en módulo $3$ es congruente a la suma de los dígitos del número y, como la suma de los dígitos no cambia al reordenarlos, este se mantiene:
$15412$ $\equiv$ $1 + 5 + 4 + 1 + 2 (3)$ $\equiv$ $13 (3)$ $\equiv$ $1 (3)$

Si los reordenamos:
$54121$ $\equiv$ $5 + 4 +1 + 2 + 1 (3)$ $\equiv$ $13 (3)$ $\equiv$ $1 (3)$

Ahora, cada vez que multiplicamos por $2$, el resto en módulo 3 también se multiplica por $2$. Esto es una propiedad de la congruencia.
Entonces, si empezamos en $1$ nos queda:

$1$ $\equiv$ $1(3)$
$2 * 1$ $\equiv$ $2 * 1(3)$ $\equiv$ $2(3)$ $\to$ $2$ $\equiv$ $2(3)$
$2 * 2$ $\equiv$ $2 * 2(3)$ $\equiv$ $4 (3)$ $\equiv$ $1(3)$
$4 * 2$ $\equiv$ $2 * 1(3)$ $\equiv$ $2(3)$

Como podemos ver, cada vez que multipliquemos por $2$ a un número congruente a $1(3)$ o congruente a $2(3)$ jamás nos dará por resultado un número congruente a $0(3)$. Como al reordenar los dígitos el resto en módulo $3$ se mantiene, ningún número que escribamos a partir de las operaciones dadas podrá ser nunca congruente a $0(3)$. Ahora, veamos a qué es congruente $9876543210$:

$9876543210$ $\equiv$ $9 + 8 + 7 + 6 + 5 + 4 + 3 + 2 + 1 + 0 (3)$ $\equiv$ $45(3)$ $\equiv$ $4 + 5 (3)$ $\equiv$ $9 (3)$ $\equiv$ $0(3)$

Dado que $9876543210$ $\equiv$ $0(3)$, comprobamos que es $imposible$ llegar a este número si empezamos en un número congruente a $1(3)$.
2  
MEDIO EQUILÁTERO?
Avatar de Usuario
marcoalonzo

FOFO 13 años - Mención-FOFO 13 años OFO - Medalla de Bronce-OFO 2024
Mensajes: 126
Registrado: Mar 18 Abr, 2023 4:52 pm
Medallas: 2

Re: Regional 2015 N1 P2

Mensaje sin leer por marcoalonzo »

Corregir si hay algún error :lol:
I)
Spoiler: mostrar
Notemos que $10=2\times 5$, entonces si elevo a $9$ ambos miembros queda que $10^{9}=(2\times 5)^{9}=2^{9}\times 5^{9}$, entonces para llegar a $10^{9}$ habría que multiplicar por $2$ y cambiar dígitos para llegar a $2^{9}\times 5^{9}$.
Empezamos por $1=2^{0}$, multiplicamos por $2$ hasta que llegamos a $512$ ($2^{9}$), y si lo cambiamos a $125$ obtenemos $3$ factores $5$ ($5^{3}$) (también podríamos reescribir el $256$ para que sea $M_{5}$, pero en su factorización hay un factor $53$ que no nos sirve). Al $125$ lo seguimos multiplicando por $2$ hasta que llegamos a $1000$ ($2^{3}$). Notemos que, como llegamos a una potencia de $10$, podemos multiplicar por cualquier número y siempre va a quedar ese número y la cantidad de ceros original; entonces podemos "pensar" a este $1000$ como un $1$, y volver a multiplicar por los factores anteriores (creamos una especie de secuencia o algoritmo):
$1000\times 2^{9}$
$512000$
$125000$
$125000\times 2^{3}$
$1000000$
$1000000\times 2^{9}$
$512000000$
$125000000$
$125000000\times 2^{3}$
$1000000000=10^{9}$
II)
Spoiler: mostrar
Notemos que $9876543210\in M_{9}$ y que para que un número sea $M_{9}$ la suma de sus cifras debe ser $M_{9}$, es decir que el paso de intercambiar los dígitos no afecta ya que la suma se mantiene. Ahora bien, tampoco se puede llegar a un $M_{9}$ multiplicando por $2$, ya que mirando módulo $9$ $9876543210\equiv 0$ entonces para llegar a $0$ todos los anteriores números deberían ser congruentes a $0$ módulo $9$ (el 0 es elemento absorbente de la mutliplicación), cosa que no es verdad ya que $1\equiv 1 \ (mod\ 9)$ (se lo puede pensar de manera muy parecida usando módulo $3$)
🔮oráculo y magia negra🔮
Responder